Prove Limit of Sequence: Sqrt(n+1)-Sqrt(n) = 0

  • Thread starter Thread starter tarheelborn
  • Start date Start date
  • Tags Tags
    Limit Sequence
Click For Summary
The limit of the sequence {Sqrt(n+1) - Sqrt(n)} is proven to be 0 by multiplying by the conjugate, resulting in the expression 1/(Sqrt(n+1) + Sqrt(n)). This transformation clarifies that as n approaches infinity, the denominator increases, leading the overall expression towards 0. To satisfy the epsilon condition, it is suggested to choose N such that 1/(Sqrt(n+1) + Sqrt(n)) < 1/m < ε for n > N. Ignoring the smaller term Sqrt(n) in the denominator is acceptable since Sqrt(n+1) is greater than Sqrt(n). The discussion concludes with the recommendation to set N as the ceiling of (1/ε^2) - 1 to ensure the limit condition is met.
tarheelborn
Messages
121
Reaction score
0

Homework Statement



Prove that the limit of the sequence {Sqrt(n+1)-Sqrt(n)} = 0.

Homework Equations




The Attempt at a Solution



I know that I must multiply by the conjugate to come up with 1/(Sqrt[n+1]-Sqrt[n]) and that the limit of this is clearly 0. I am having trouble solving this equation in terms of epsilon.
 
Physics news on Phys.org
Be careful of the sign; multiplying by the conjugate gives you 1/{sqrt(n+1) + sqrt(n)}. For all ε > 0, there exists 1/m < ε for some positive integer m. (Why?) How should you choose N so that 1/{sqrt(n+1) + sqrt(n)} < 1/m < ε whenever n > N?
 
Ah, yes, the sign. Thank you! Now... To choose N, can I ignore the sqrt(n) part of the denominator since sqrt(n+1) > sqrt(n) so, being in the denomominator, the number is smaller? So could I let N = (1/epsilon^2) - 1?
 
That should work fine; by convention we let N be an integer, so you can set N to be the ceiling of what you have.
 
Thank you!
 
Question: A clock's minute hand has length 4 and its hour hand has length 3. What is the distance between the tips at the moment when it is increasing most rapidly?(Putnam Exam Question) Answer: Making assumption that both the hands moves at constant angular velocities, the answer is ## \sqrt{7} .## But don't you think this assumption is somewhat doubtful and wrong?

Similar threads

  • · Replies 17 ·
Replies
17
Views
2K
  • · Replies 12 ·
Replies
12
Views
2K
  • · Replies 9 ·
Replies
9
Views
3K
  • · Replies 7 ·
Replies
7
Views
1K
  • · Replies 4 ·
Replies
4
Views
2K
  • · Replies 13 ·
Replies
13
Views
2K
  • · Replies 1 ·
Replies
1
Views
2K
  • · Replies 1 ·
Replies
1
Views
2K
  • · Replies 4 ·
Replies
4
Views
2K
  • · Replies 17 ·
Replies
17
Views
2K